LSAT and Law School Admissions Forum

Get expert LSAT preparation and law school admissions advice from PowerScore Test Preparation.

 ssnasir
  • Posts: 22
  • Joined: Feb 22, 2020
|
#74719
Hi there,

The author's argument is that the practice in not environmentally sound and we need an answer choice that proves that, but I have seen on other questions where the contrapositive is the answer. The contrapositive of D would be that an environmental practice cannot be considered sound :arrow: the costs do not outweigh the benefits (in which case the cost are more than the benefit) which is sort of whats happening in the stimulus too because sphagnum has benefits but its use is contributing to a cost (depleting oxygen). So, I'm still a little lost with D

thank you!
 Adam Tyson
PowerScore Staff
  • PowerScore Staff
  • Posts: 5153
  • Joined: Apr 14, 2011
|
#74733
Two issues with your analysis here, ssnasir, and the first is that we are NOT looking for proof of the conclusion, but only an answer that "most helps" the argument. This is a Strengthen question, not a Justify the Conclusion.

Second, let's think about the impact of the contrapositive of answer D. The necessary condition would be the benefits not outweighing the costs, and that means either the costs outweigh the benefits or else they are equal. So, if the practice is not environmentally sound, benefits don't outweigh costs. What does that do for our argument, where the author is trying to show that the practice is not sound? The argument has showed us a major cost of using sphagnum moss, but is there any evidence that the cost is not outweighed by some benefit? All it tells us is that there is a cost - it never compares that cost to any benefit, so we cannot know if the Necessary Condition (benefits not outweighing costs) has been met.

And what if it had been? Would knowing that the Necessary Condition is met do anything to strengthen a claim that the Sufficient Condition had been met? That sounds an awful lot like a Mistaken Reversal, doesn't it? If it's a flaw when an LSAT author does it, it's also a flaw for us to try to use it to help an argument!

Answer B goes right to the point here. If it is true that reducing the oxygen supply proves that a practice is not sound, and if using this moss reduces the oxygen supply, then using the moss cannot be sound. It's a Strengthen question, but that answer is strong enough to Justify the Conclusion, and nothing could help more than that! That HAS to be the best answer choice! No matter how helpful you may find answer D, or any other answer choice, you are supposed to pick the one that does the MOST to help the argument, and answer B is all that and a bag of sphagnum moss.
 ssnasir
  • Posts: 22
  • Joined: Feb 22, 2020
|
#74742
Adam Tyson wrote:Two issues with your analysis here, ssnasir, and the first is that we are NOT looking for proof of the conclusion, but only an answer that "most helps" the argument. This is a Strengthen question, not a Justify the Conclusion.

Second, let's think about the impact of the contrapositive of answer D. The necessary condition would be the benefits not outweighing the costs, and that means either the costs outweigh the benefits or else they are equal. So, if the practice is not environmentally sound, benefits don't outweigh costs. What does that do for our argument, where the author is trying to show that the practice is not sound? The argument has showed us a major cost of using sphagnum moss, but is there any evidence that the cost is not outweighed by some benefit? All it tells us is that there is a cost - it never compares that cost to any benefit, so we cannot know if the Necessary Condition (benefits not outweighing costs) has been met.

And what if it had been? Would knowing that the Necessary Condition is met do anything to strengthen a claim that the Sufficient Condition had been met? That sounds an awful lot like a Mistaken Reversal, doesn't it? If it's a flaw when an LSAT author does it, it's also a flaw for us to try to use it to help an argument!

Answer B goes right to the point here. If it is true that reducing the oxygen supply proves that a practice is not sound, and if using this moss reduces the oxygen supply, then using the moss cannot be sound. It's a Strengthen question, but that answer is strong enough to Justify the Conclusion, and nothing could help more than that! That HAS to be the best answer choice! No matter how helpful you may find answer D, or any other answer choice, you are supposed to pick the one that does the MOST to help the argument, and answer B is all that and a bag of sphagnum moss.
Thank you so much!
User avatar
 ToadKing
  • Posts: 18
  • Joined: Jan 17, 2021
|
#88858
Answer (A) weakens the gardeners' argument, but doesn't strengthen the author's argument. For that reason, it is incorrect, right?

Based on the stimulus, the gardeners mistakenly assume that "no chemical additives" and "renewable resource" are sufficient to prove that a practice is environmentally sound. Answer (A) pokes a hole in that by stating that "no chemical additives" and "renewable resource" is insufficient to prove that something is environmentally sound. However, to strengthen the author's argument we need to support their conclusion instead of weakening the gardener's argument, right?

Thanks!
 Rachael Wilkenfeld
PowerScore Staff
  • PowerScore Staff
  • Posts: 1358
  • Joined: Dec 15, 2011
|
#89009
Right, Toadking. We an answer choice that gets us all the way to the conclusion--that using peat moss as soil conditioner is not environmentally sound. Answer choice (A) doesn't really weaken the argument, but it doesn't strengthen it either. It says that there is a possibility that the conclusion is true, but it doesn't actually provide information to strengthen that conclusion.

Hope that helps!

Get the most out of your LSAT Prep Plus subscription.

Analyze and track your performance with our Testing and Analytics Package.